LSAT and Law School Admissions Forum

Get expert LSAT preparation and law school admissions advice from PowerScore Test Preparation.

 Administrator
PowerScore Staff
  • PowerScore Staff
  • Posts: 8917
  • Joined: Feb 02, 2011
|
#23681
Complete Question Explanation

Flaw in the Reasoning-SN. The correct answer choice is (B)

The stimulus concludes that more people believe elected officials should resign if indicted that believe they should resign if convicted, and argues that on the basis that 50% of people believe an official should resign if indicted, whereas 35% believe one should resign only if convicted.

If you keep your Conditional wits about you, you can escape the somewhat confusing nature of the stimulus by simply observing that since the 50% is accompanied by an "if," while the 35% is accompanied by an "only if," the percentages cannot be compared because they are about logically different types of conditions.

If not, you will need to diagram the stimulus so that you can clarify the issue:
  • Premises: ..... 50% believe "Indicted ..... :arrow: ..... Should Resign"

    ..... ..... 35% believe "Resign ..... :arrow: ..... Convicted."

    Conclusion: More believe that a politician should resign if indicted than believe a politician should resign if convicted.
First of all, you should recognize once again that you cannot make the comparison in the conclusion, because you do not have information about what people believe should occur if someone is convicted, because conviction was never presented as a sufficient condition.

Secondly, that conclusion is absurd, since everyone who has been convicted has by definition been indicted (that is common knowledge and you are supposed to realize it). Everyone who believes a politician should resign if indicted by definition believes that the politician should resign if convicted.

The LSAT test-writers are simply using numbers that seem to accompany their reasoning quite well in the hopes that you will have a hard time disbelieving their fundamentally absurd and flawed reasoning.

However, the easiest way to get this question correct is simply to observe the illogical comparison of a sufficient condition to a necessary position, and proceed to identify that flaw.

Answer choice (A): This choice references an overgeneralization, but the stimulus involved a necessary-sufficient error. This choice is wrong.

Answer choice (B): This is the correct answer choice. Simply focusing on the "if" and "only if" should be sufficient for you to notice that this choice is very probably the correct response, even if your analysis is no deeper.

Answer choice (C): The argument is not based on the ambiguity of any of the terms. You should avoid choosing such a response when you simply find the stimulus confusing. Frequently, confusing stimuli are accompanied by choices about circular reasoning, ambiguous terms, and impossible reasoning in the hopes that confused test-takers will basically choose the suicide option for such questions. If you are confused, eliminate these types of choices. This choice is incorrect.

Answer choice (D): It is unclear what the third specific belief is, unless it concerns comparing the numbers of people who hold certain beliefs. It is actually not a flaw to use information about two different beliefs to make conclusions about certain third beliefs, as long as the information is used in a consistent, logical manner. Since it is not the goal, but rather the method, that is flawed, this choice is wrong.

Answer choice (E): The argument contains no contradiction, and this choice is for the testers who could not understand the 50% and 35%. If you chose this answer, you probably realized that the conclusion had to be false, because if 50% of people believe politicians should resign if indicted, then at least 50% of people have that same belief about conviction. However, you probably did not realize that the 35% is described by "only if," so is not contradictory.
 desmail
  • Posts: 50
  • Joined: Jul 05, 2011
|
#4434
Hi,

I can't seem to figure out why answer choice (D) is wrong. I can see how (B) is correct, but (D) also claims that the author draws a conclusion (that only if=if, mixing up the sufficient and necessary conditions) based on 2 responses from a survey.

So the author does seem to be drawing an irrelevant conclusion by adding another group (the "if convicted" group) based off of two responses from a survey (the 35% "only if convicted" and the 50% "if indicted").

Any comments on how I could prove this answer choice to be wrong?

Thanks!
 Adam Tyson
PowerScore Staff
  • PowerScore Staff
  • Posts: 5153
  • Joined: Apr 14, 2011
|
#4435
Hi desmail. Let's take a look at how this conditional argument works.

First, we have 50% believing that if an elected official is indicted, they should resign. That's something like "I -> R (50%)".

Next, we have 35% who believe they should resign ONLY IF convicted. That "only if" indicates a necessary condition, so what we get is "R -> C (35%)".

The conclusion then attempts to compare the two groups, but does so by making a Mistaken Reversal of the second group - something like "I -> R is larger than C -> R". See how that second half of the diagram reverses the sufficient and necessary conditions on the original premise? There's the real problem.

We also might have seen this put in terms of numbers and percentages - the author failed to consider that all the members of the first group would also likely favor resignation in the face of conviction, or that the two should be added together (or at least overlap).

So, what's wrong with answer D, you ask? The argument doesn't draw any specific conclusion about any particular belief, it just improperly compares the two belief groups that were already established to see which group is bigger. If you liked D, ask yourself what did the author conclude about any one specific belief? You would be looking for something like "the members of the first group do not believe X" or "the members of the second group are incorrect."

Hope that helped!

Adam M. Tyson
PowerScore LSAT Instructor
 desmail
  • Posts: 50
  • Joined: Jul 05, 2011
|
#4441
Got it! Thanks!
 GLMDYP
  • Posts: 100
  • Joined: Aug 19, 2013
|
#10387
Hi Powerscore!
I'm just wondering why (C) is not the right answer? And how can (B) fit into this?
Thanks!
 Ron Gore
PowerScore Staff
  • PowerScore Staff
  • Posts: 220
  • Joined: May 15, 2013
|
#10407
GLMDYP,

Your task in this Method of Reasoning - Flaw question is to find the answer that most accurately describes the logical flaw present in the stimulus. The first step is to understand the argument and then prephrase. Here, the argument proceeds:

Premise: 50% believe Indicted ---> Resign

Premise: 35% believe Resign ----> Convicted

Conclusion: more people believe elected officials should resign if indicted than believe that they should resign if convicted.

The flaw in this argument is that the conditional statements don't match up. The conclusion treats the second premise as if it were Convicted ---> Resign.

When you see an error in conditional reasoning and you are in a Method of Reasoning - Flaw question, search through the answers for one discussing being sufficient or required, i.e., the type of language used in conditional reasoning. Here, choice (B) presents that type of language, describing a Mistaken Reversal, and is correct.

For choice (C) to be correct, you would have to see in the stimulus some term or phrase being given more than one meaning. There is no such usage in this stimulus, so (C) is incorrect.

Ron
 ellenb
  • Posts: 260
  • Joined: Oct 22, 2012
|
#10584
Dear Powerscore,

In this stimulus, I did not understand why B is the correct answer, I picked A, since I thought that the stimulus draws a conclusion about the general public (people) based on survey results only. So, that is why I picked A. However, I am not sure how in this stimulus the sufficient condition is confused with a required condition, is it because in the second statement the stimulus has "35 percent believe should resign-->only if they are convicted of a crime" compared to the first statement which says "if indicted for a crime-->50% believe that elected officials should resign" So, essentially the conclusion is comparing a sufficient with a necessary.

Please let me know whether I sort of was close to why B is the correct answer.

Thanks

Ellen
 David Boyle
PowerScore Staff
  • PowerScore Staff
  • Posts: 836
  • Joined: Jun 07, 2013
|
#10603
ellenb wrote:Dear Powerscore,

In this stimulus, I did not understand why B is the correct answer, I picked A, since I thought that the stimulus draws a conclusion about the general public (people) based on survey results only. So, that is why I picked A. However, I am not sure how in this stimulus the sufficient condition is confused with a required condition, is it because in the second statement the stimulus has "35 percent believe should resign-->only if they are convicted of a crime" compared to the first statement which says "if indicted for a crime-->50% believe that elected officials should resign" So, essentially the conclusion is comparing a sufficient with a necessary.

Please let me know whether I sort of was close to why B is the correct answer.

Thanks

Ellen
Hello Ellen,

A is not really about the general public, so much as trying to do math about a section of the public (those polled about elected officials' resigning).
Yes, you were sort of close, in that "more people believe that elected officials should resign if indicted than believe that they should resign if convicted" misinterprets "35 percent believe that elected officials should resign only if they are convicted of a crime" as saying "only if they are convicted" means the same as "if they are convicted". But it isn't, as the first, "only if", has "convicted" as a necessary condition, but the second, "if", has "convicted" as a sufficient condition.

Hope that helps,
David
 Haleyeastham
  • Posts: 33
  • Joined: Aug 03, 2015
|
#19710
Can you please explain correct answer choice B to me?

Thank you!
 David Boyle
PowerScore Staff
  • PowerScore Staff
  • Posts: 836
  • Joined: Jun 07, 2013
|
#19716
Haleyeastham wrote:Can you please explain correct answer choice B to me?

Thank you!
Hello,

Answer B is about mistaken reversal, basically, and confusing sufficient with necessary ("required"). The first sentence of the stimulus says, 35 percent want resignation ONLY IF conviction. "Only if" is a necessary indicator.
But the second sentence says. "resign IF convicted". There, "convicted" is the sufficient condition, as shown by the placement of the "if". Therefore, the second sentence does a mistaken reversal on what the first sentence says.

David

Get the most out of your LSAT Prep Plus subscription.

Analyze and track your performance with our Testing and Analytics Package.